Difference between revisions of "MIE 2016/Day 1/Problem 2"

(Created page with "===Problem 2=== The following system has <math>k</math> integer solutions. We can say that: <math>\begin{cases}\frac{x^2-2x-14}{x}>3\\\\x\leq12\end{cases}</math> (a) <math>...")
 
(Added solution)
Line 1: Line 1:
===Problem 2===
+
==Problem 2==
 
The following system has <math>k</math> integer solutions. We can say that:
 
The following system has <math>k</math> integer solutions. We can say that:
  
Line 16: Line 16:
  
  
===Solution 2===
+
==Solution 2==
 +
<math>\frac{x^2-2x-14}{x}>3</math>
  
 +
<math>\frac{x^2-2x-14}{x}-\frac{3x}{x}>0</math>
 +
 +
<math>\frac{x^2-5x-14}{x}>0\begin{cases}x^2-5x-14>0\\x>0\end{cases}</math>
 +
 +
<math>\left.\begin{array}{l}x > 0\\x > 7\\x < -2\end{array}\right\}x\in(-2,0)\cup(7,+\infty)</math>
 +
 +
 +
Adding the other interval we get
 +
 +
 +
<math>x\in(-2,0)\cup(7,12]</math>
 +
 +
 +
If <math>k</math> is the number of integer solutions, then <math>k=7</math>. <math>\boxed{D}</math>
  
  
 
===See Also===
 
===See Also===

Revision as of 19:29, 8 January 2018

Problem 2

The following system has $k$ integer solutions. We can say that:

$\begin{cases}\frac{x^2-2x-14}{x}>3\\\\x\leq12\end{cases}$


(a) $0\leq k\leq 2$

(b) $2\leq k\leq 4$

(c) $4\leq k\leq6$

(d) $6\leq k\leq8$

(e) $k\geq8$


Solution 2

$\frac{x^2-2x-14}{x}>3$

$\frac{x^2-2x-14}{x}-\frac{3x}{x}>0$

$\frac{x^2-5x-14}{x}>0\begin{cases}x^2-5x-14>0\\x>0\end{cases}$

$\left.\begin{array}{l}x > 0\\x > 7\\x < -2\end{array}\right\}x\in(-2,0)\cup(7,+\infty)$


Adding the other interval we get


$x\in(-2,0)\cup(7,12]$


If $k$ is the number of integer solutions, then $k=7$. $\boxed{D}$


See Also